Last visit was: 26 Apr 2024, 07:51 It is currently 26 Apr 2024, 07:51

Close
GMAT Club Daily Prep
Thank you for using the timer - this advanced tool can estimate your performance and suggest more practice questions. We have subscribed you to Daily Prep Questions via email.

Customized
for You

we will pick new questions that match your level based on your Timer History

Track
Your Progress

every week, we’ll send you an estimated GMAT score based on your performance

Practice
Pays

we will pick new questions that match your level based on your Timer History
Not interested in getting valuable practice questions and articles delivered to your email? No problem, unsubscribe here.
Close
Request Expert Reply
Confirm Cancel
SORT BY:
Kudos
Tags:
Show Tags
Hide Tags
VP
VP
Joined: 30 Jan 2016
Posts: 1232
Own Kudos [?]: 4560 [15]
Given Kudos: 128
Send PM
Verbal Forum Moderator
Joined: 08 Dec 2013
Status:Greatness begins beyond your comfort zone
Posts: 2101
Own Kudos [?]: 8811 [1]
Given Kudos: 171
Location: India
Concentration: General Management, Strategy
GPA: 3.2
WE:Information Technology (Consulting)
Send PM
Manager
Manager
Joined: 21 Apr 2018
Posts: 60
Own Kudos [?]: 41 [1]
Given Kudos: 82
Location: India
GMAT 1: 710 Q50 V35
GMAT 2: 750 Q49 V42
Send PM
Intern
Intern
Joined: 30 Sep 2018
Posts: 18
Own Kudos [?]: 90 [1]
Given Kudos: 115
Send PM
Re: Council member: The profits of downtown businesses will increase if [#permalink]
1
Kudos
nightblade354

Although I got this question right, I have some reservation regarding answer choice B. I see the diagram that you did, thanks for sharing and it's very helpful. I'm having trouble seeing why the last sentence can be implemented as part of the flow in the diagram when we are not given any references explaining the relationship btw traffic congestion level and downtown biz’ profits. To put it another way, I’m having trouble understanding why we can choose answer choice B when we do not know if traffic congestion is / has been alleviated. Apology for any poor wording here. Hopefully my question makes sense.

Thanks for you help!
Manager
Manager
Joined: 17 Oct 2016
Posts: 227
Own Kudos [?]: 426 [1]
Given Kudos: 49
Location: India
Concentration: General Management, Healthcare
GPA: 3.05
WE:Pharmaceuticals (Health Care)
Send PM
Re: Council member: The profits of downtown businesses will increase if [#permalink]
1
Bookmarks
Council member: The profits of downtown businesses will increase if more consumers live in the downtown area, and a decrease in the cost of living in the downtown area will guarantee that the number of consumers living there will increase. However, the profits of downtown businesses will not increase unless downtown traffic congestion decreases.

If all the council member’s statements are true, which one of the following must be true?

(A) If downtown traffic congestion decreases, the number of consumers living in the downtown area will increase. WRONG - No correlation between traffic congestion and number of consumers living in downtown area.

(B) If the cost of living in the downtown area decreases, the profits of downtown businesses will increase. Correct - Implied in highlighted sentence

(C) If downtown traffic congestion decreases, the cost of living in the downtown area will increase.WRONG - No correlation between traffic congestion and cost of living.

(D) If downtown traffic congestion decreases, the cost of living in the downtown area will decrease. Wrong - No Correlation

(E) If the profits of downtown businesses increase, the number of consumers living in the downtown area will increase. WRONG - The cause and effect are flipped/reversed
Manager
Manager
Joined: 21 Apr 2018
Posts: 60
Own Kudos [?]: 41 [1]
Given Kudos: 82
Location: India
GMAT 1: 710 Q50 V35
GMAT 2: 750 Q49 V42
Send PM
Re: Council member: The profits of downtown businesses will increase if [#permalink]
1
Bookmarks
Hi nightblade354,

Could you please look at the option choices for this question? Won't statement following however has any impact on answer choice.
Current Student
Joined: 31 Jul 2017
Status:He came. He saw. He conquered. -- Going to Business School -- Corruptus in Extremis
Posts: 1734
Own Kudos [?]: 5742 [0]
Given Kudos: 3054
Location: United States (MA)
Concentration: Finance, Economics
Send PM
Council member: The profits of downtown businesses will increase if [#permalink]
Expert Reply
Wonderwoman31,

You forgot to tag me! But no matter. Here is a diagram that I hope helps you and everyone else who may struggle on this. By the way, this is heavy with LSAT Lawgic; the GMAT won't usually force you to use sufficient and necessary conditions to answer questions (though this technique can really help)!

Decrease in Cost of Living Down Town ---> More Customers Living Down Town ---> Profits of Businesses Down Town Increase ---> Down Town Traffic Congestion Decreases

The answer to the question will be something that follows our path, and indeed the answer does
Current Student
Joined: 31 Jul 2017
Status:He came. He saw. He conquered. -- Going to Business School -- Corruptus in Extremis
Posts: 1734
Own Kudos [?]: 5742 [0]
Given Kudos: 3054
Location: United States (MA)
Concentration: Finance, Economics
Send PM
Re: Council member: The profits of downtown businesses will increase if [#permalink]
Expert Reply
Wonderwoman31,

That is exactly what my diagram shows. If the first items occurs, then the second occurs. If the second occurs, then the third occurs. And so one and so forth.
Retired Moderator
Joined: 19 Oct 2018
Posts: 1878
Own Kudos [?]: 6296 [0]
Given Kudos: 704
Location: India
Send PM
Re: Council member: The profits of downtown businesses will increase if [#permalink]
There are 2 independent scenarios
1. CL decrease>>> Number of people increase>>>>profit increase
2. TC decrease>>> Profit increase

A. Not given
B. statement 1 clearly shows that
C. Not given
D. Not given
E. Reverse case
Manager
Manager
Joined: 21 Jul 2018
Posts: 153
Own Kudos [?]: 434 [0]
Given Kudos: 80
Location: United States
Concentration: General Management, Social Entrepreneurship
Send PM
Council member: The profits of downtown businesses will increase if [#permalink]
Akela wrote:
Council member: The profits of downtown businesses will increase if more consumers live in the downtown area, and a decrease in the cost of living in the downtown area will guarantee that the number of consumers living there will increase. However, the profits of downtown businesses will not increase unless downtown traffic congestion decreases.

If all the council member’s statements are true, which one of the following must be true?

(A) If downtown traffic congestion decreases, the number of consumers living in the downtown area will increase.
(B) If the cost of living in the downtown area decreases, the profits of downtown businesses will increase.
(C) If downtown traffic congestion decreases, the cost of living in the downtown area will increase.
(D) If downtown traffic congestion decreases, the cost of living in the downtown area will decrease.
(E) If the profits of downtown businesses increase, the number of consumers living in the downtown area will increase.



The symbolic logic for the solution as follows:

P1: The profits of downtown businesses will increase if more consumers live in the downtown area.

↑DTC (increase # of downtown area consumers) ⇢ ↑ π

P2: and a decrease in the cost of living in the downtown area will guarantee that the number of consumers living there will increase.

↓ COL (decrease cost of living) ⇢ ↑DTC

P3: However, the profits of downtown businesses will not increase unless downtown traffic congestion decreases.

~ ↓ T (unless traffic decreases) ⇢ ~↑π

To find a conclusion, we combine the premises above to get:

↑DTC ⇢ ↑ π ................................................(From P1)
↓ COL ⇢ ↑DTC ............................................(From P2)
↑π ⇢ ↓ T ......................................................(From ~P3)
↓ COL ⇢ ↑DTC ⇢ ↑ π ⇢ ↓ T ........................(From P1+P2+~P3)


Now, comparing this against our answer choices, we have:

A) If downtown traffic congestion decreases, the number of consumers living in the downtown area will increase.
Not given....basically, concluding that ↓ T ⇢ ↑DTC, which is not matching any premise above.

(B) If the cost of living in the downtown area decreases, the profits of downtown businesses will increase.
↓ COL ⇢ ↑DTC ⇢ ↑ π ⇢ ↓ T

(C) If downtown traffic congestion decreases, the cost of living in the downtown area will increase.
This is a mistaken reversal of our combined statement (P1+P2+~P3): ↓ COL ⇢↑DTC ⇢ ↑ π ⇢ ↓ T.
Mistaken reversal: ↓ T ⇢ ↑ π ⇢↑DTC ⇢↓ COL.

(D) If downtown traffic congestion decreases, the cost of living in the downtown area will decrease.
Same as C. Also a mistaken reversal. Not necessarily true.

(E) If the profits of downtown businesses increase, the number of consumers living in the downtown area will increase.
Same as C and D. A mistaken reversal.



Current Student
Joined: 31 Jul 2017
Status:He came. He saw. He conquered. -- Going to Business School -- Corruptus in Extremis
Posts: 1734
Own Kudos [?]: 5742 [0]
Given Kudos: 3054
Location: United States (MA)
Concentration: Finance, Economics
Send PM
Council member: The profits of downtown businesses will increase if [#permalink]
Expert Reply
snz201519 wrote:
nightblade354

Although I got this question right, I have some reservation regarding answer choice B. I see the diagram that you did, thanks for sharing and it's very helpful. I'm having trouble seeing why the last sentence can be implemented as part of the flow in the diagram when we are not given any references explaining the relationship btw traffic congestion level and downtown biz’ profits. To put it another way, I’m having trouble understanding why we can choose answer choice B when we do not know if traffic congestion is / has been alleviated. Apology for any poor wording here. Hopefully my question makes sense.

Thanks for you help!


Whenever we have a situation where we have X --> Y --> Z, it has to be that X --> Z; this is how the diagram connects. Let me give you an example: If you run really fast, you will win the race. If you win the race, you will be a hero. Therefore, if you run really fast, you will be a hero. The same principle applies, per logic. The sufficient condition triggers the necessary every time. As a result, a statement (Y in this case) can act as a necessary condition for X and a sufficient condition for Z. When X is triggered, Y has to and it follows that Z has to as well.

I hope that clears it up! If not, please ask.
Intern
Intern
Joined: 27 Jul 2021
Posts: 10
Own Kudos [?]: 0 [0]
Given Kudos: 80
Send PM
Re: Council member: The profits of downtown businesses will increase if [#permalink]
How can you conclude B is the ans when there is no correlation between More people living in Downtown equates to less congestion in traffic. We are extrapolating here by everyday scenario and not because of question thread logic. B could have been right if Last sentence was not there.
GMAT Club Bot
Re: Council member: The profits of downtown businesses will increase if [#permalink]
Moderators:
GMAT Club Verbal Expert
6921 posts
GMAT Club Verbal Expert
238 posts
CR Forum Moderator
832 posts

Powered by phpBB © phpBB Group | Emoji artwork provided by EmojiOne